Rutgers University Graduate Program in Mathematics
Rutgers University Graduate Program in Mathematics
Rutgers University Graduate Program in Mathematics
X
X
f d =
f d
(bj aj ) = (U ) (1 + /2)(F ) .
U
aj
j=1
j=1
Since
Z
Z
Z
f d f d
U
U F
f d ,
c
U F
f d .
c
Z
Hence
f d (/2)(F ) for every such open set U containing F .
U F c
Since (F ) > 0 this last inequality is aR contradiction. Indeed, since f is
integrable, we can choose such a U with U F c |f | d < (/2)(F ).
2. Use contour integration to evaluate
Z
1
dx.
(1 + x2 )2
0
Be clear about any computation of residues and about any computations of
limits of integrals.
1
. Then the poles of f are at z = i, i and
(1 + z 2 )2
are of order 2. Let R > 1. and consider the counter clockwise closed path
consisting of
Solution. Let f (z) =
n 2 o
= 2i
= .
3
(z + i) z=i
2
2
On the other hand, if R > 2 there is a constant C > 0 such that |f (z)|
4
C/R
for z on 2 . Since the length of 2 is R, this gives the estimate
R
|f
(z)dz|
C/R3 . Therefore
2
Z
0
Z
Z
1
1
1 R
1
1
dx =
dx = lim
dx
2
2
2
2
R 2 R (1 + x2 )2
(1 + x )
2 (1 + x )
Z
Z
1
f (z) dz
f (z) dz = .
= lim
2 R
4
2
3. Let S9 denote the symmetric group on {1, 2, ..., 9} and let S9 be given
(in table form) by
1 2 3 4 5 6 7 8 9
=
.
9 5 8 1 7 2 6 3 4
As usual C(), the centralizer of in S9 , is defined to be C() = {
S9 | = }. Find |C()| and justify your answer.
First Solution. In cycle form = (194)(2576)(38). S9 acts on itself
by conjugation and C() is the stabilizer of . The orbit of consists of
9!(2!3!1!)/(3!4!2!) = 9!/(24) elements. This is because there are 9!/(3!4!2!)
partitions of {1, . . . , 9} into subsets of cardinalities 3, 4, and 2 and there are
(k 1)! distinct k-cycles permuting a set of k elements. Hence |C()| =
|S9 |/|S9 | = (9!)/(9!/24) = 24.
Second Solution. In cycle form = (194)(2576)(38). S9 acts on itself
by conjugation and C() is the stabilizer of . If S9 then 1 =
( (1) (9) (4))( (2) (5) (7) (6))( (3) (8)). Thus if 1 = then is
determined by the choices of (1) (3 possibilities), (2) (4 possibilities), and
(3) (2 possibilities). This gives a total of 3 4 2 = 24 elements in C().
First DayPart II: Answer three of the following questions. If you
work on more than three questions, indicate clearly which three
should be graded.
4. Recall that a set X in a topological space is called a G set when it is a
countable intersection of open sets, and it is called an F set when it is a
(Uk Ac ) 1/(2k) .
Since
(Uk Fkc ) = (Uk Ac ) + (A Fkc ) < 1/k ,
the assertion is proved.
5. Let f be analytic on the unit disc D, and assume that |f (z)| < 1 for all
z D. Prove that if there exist two distinct points a and b in the disc which
are fixed points, that is, f (a) = a and f (b) = b, then f (z) = z for all z D.
az
for z D be the Mobius transform. Then a
1a
z
is an automorphism of D with inverse a . Consider F (z) = a f a (z).
Then F (0) = 0 and
ab
ab
F
=
6= 0 .
(?)
1a
b
1a
b
Solution. Let a (z) =
has 16 distinct 5-Sylow subgroups. Since these subgroups are cyclic of prime
order, their pairwise intersections are trivial, so in this case G has at most
80 16 4 = 16 elements whose orders are powers of 2. These elements form
only one Sylow 2-subgroup, and therefore that subgroup is normal. Hence G
is not simple.
7. Let X and Y be topological spaces and f : X Y and g : X Y be
continuous functions. Prove that if Y is a Hausdorff space then {x X :
f (x) = g(x)} is closed.
Solution. let E = {x X : f (x) = g(x)}. Suppose z 6 E; we show that
there is a neighborhood of z disjoint from E. Since f (z) 6= g(z) there are
neighborhoods Nf of f (z) and Ng of g(z) that are disjoint. By continuity of
f and g, f 1 (Nf ) and g 1 (Ng ) are neighborhoods of z. Let N = f 1 (Nf )
g 1 (Ng ). Then N is a neighborhood of z. Now we claim that N E = .
Let w N . Then f (w) Nf and g(w) Ng and the disjointness of Nf and
Ng implies f (w) 6= g(w) so w 6 E.
8. Let (fn ) be a sequence of nonnegative integrable functions on [0, 1] converging almost everywhere to a function f (x). Prove that if
Z
Z
f d
fn d =
lim
n
[0,1]
[0,1]
then
lim fn = f
in L1 [0, 1].
First Solution. Since fn 0, we have 0 min{fn , f } f . Since f
L1 [0, 1] and min{fn , f } f pointwise as n , the Lebesgue Dominated
Convergence Theorem implies that
Z
Z
lim
min{fn , f } d =
f d .
n
[0,1]
[0,1]
[0,1]
[0,1]
[0,1]
Second Solution. Set X = [0, 1] and let > 0R be given. Since f is integrable
and nonnegative, there exists > 0 such that E f d < for any measurable
set E X with (E) < . Since (X) < , Egorovs Theorem implies
that there exists a measurable set E with (E) < and {fn } converging to
f uniformly on X \ E. Since (X \ E) < , the uniform convergence on
X \ E and the assumed convergence of the integrals implies that there exits
an integer N such that
Z
Z
|fn f | d < and (fn f ) d < for all n N .
(?)
X\E
X\E
3 .
From this estimate we obtain
Z
Z
|fn | + |f | d 4 .
|fn f | d
E
Thus
|fn f | d
X
|fn f | d +
X\E
|fn f | d 5
E
for all n N . Since > 0 was arbitrary, this proves the convergence in L1 .
9. Let A and B be commuting 8 by 8 diagonalizable matrices over the real
numbers with characteristic polynomials
det(A I) = ( 1)3 ( 3)5
and
det(B I) = 2 ( 4)6 .
Suppose the minimum polynomial of A B is
(2 1)(2 9) .
Find the dimension of the vector space of all 8 by 8 real matrices that commute with both A and B.
Solution. Let V1 , V3 be the eigenspaces with eigenvalues = 1 and = 3
for A. Then dim V1 = 3 and dim V3 = 5 since A is diagonalizable. Likewise,
let W0 , W4 be the eigenspaces with eigenvalues = 0 and = 4 for B. Then
dim W0 = 2 and dim W4 = 6 since B is diagonalizable. Let Vi,j = Vi Wj for
i = 1, 3 and j = 0, 4. Since B commutes with A,
R8 = V1,0 V1,4 V3,0 V3,4
and A and B act by the scalars i and j, respectively, on Vi,j . We have
dim V1,0 + dim V3,0 = 2 ,
dim V1,0 + dim V1,4 = 3 ,
(?)
RUTGERS UNIVERSITY
GRADUATE PROGRAM IN MATHEMATICS
Written Qualifying Examination
January 2011, Day 2
Second DayPart I: Answer each of the following three questions
1. Let f (x) be a function on [0, 1] and suppose that f 0 (x) is defined for all
0 x 1. Prove that f 0 (x) is a measurable function.
Solution. Take a differentiable extension of f (x) to the right of x = 1. For
example, set f (1 + a) = f (1) + af 0 (1) for a > 0. Set
1
n (x) = n f x +
f (x) .
n
Each n (x) is continuous and therefore measurable. Since f 0 (x) = limn n (x)
it is measurable.
2. Prove that all the roots of z 7 5z 3 + 12 = 0 lie in {z C : 1 |z| 2}.
Solution. Let f (z) = z 7 5z 3 + 12, g(z) = z 7 , and h(z) = 12. On |z| = 1,
|f (z) h(z)| = |z 7 5z 3 | 6 < 12 = |h(z)| .
Therefore f (z) has no zeros in |z| < 1 by Rouches theorem, since h(z) has
no zeros there. On |z| = 2,
|f (z) g(z)| = |5z 3 12| 5 23 + 12 = 26 < 27 = |g(z)| .
Therefore f (z) has 7 zeros in |z| 2 by Rouches theorem, since g(z) has 7
zeros there (counting multiplicities). By the first part, the zeros of f (z) all
have modulus greater than 1, and since f (z) has degree 7, these are all of its
zeros.
3. Are the quotient rings Z[x]/(x3 +1) and Z[x]/(x3 +2x2 +x+1) isomorphic?
Provide full justification for your answer.
Solution. The rings are not isomorphic. In fact, the first ring contains zero
divisors since x3 + 1 = (x + 1)(x2 x + 1) is not irreducible in Z[x]. For the
second ring, note that 1 is not a root of p(x) = x3 + 2x2 + x + 1. Since
p(x) is monic and the product of its (complex) roots is the constant term 1,
it follows that p(x) has no integer roots. Hence p(x) is irreducible in Z[x],
since any factorization of it would include at least one linear factor (because
p(x) has degree 3). Since Z[x] is a unique factorization domain, it follows
that p(x) is prime. Thus Z[x]/(p(x)) is an integral domain. This proves that
the rings are not isomorphic.
Second DayPart II: Answer three of the following questions. If
you work on more than three questions, indicate clearly which
three should be graded.
4. Find the Laurent expansion of f (z) = (1 z 2 )e1/z around z = 0. Compute
the residue at 0.
Solution. The Laurent series for e1/z is given by
1/z
X
1
.
=
n!z n
n=0
Hence
X
X
1
1
n2 + 3n + 1 n
2
f (z) =
z .
=
z
z
+
n
n2
n!z
n!z
(n
+
2)!
n=0
n=0
The coefficient of z 1 in the series is (12 + 3 1 + 1)/3!. Thus Resf (0) = 5/6.
5. Let f be a complex-valued measurable function on R. Let be Lebesgue
measure and suppose that for each g L2 (), the function f g L1 (). Show
that f L2 ().
Solution. For each positive integer N define
Z
2
FN = {g L () :
|f g| d N } .
10
L () =
FN .
(?)
N =1
2
|hf |d < .
(??)
khk2 =1
R
Hence the linear functional g 7 F (g) = f g d, for g L2 (), is bounded
with bound M . By the Riesz Representation
Theorem, there exists a function
R
2
L () such that F (g) = g d for all g L2 (). Hence f = almost
everywhere, so f L2 ().
(??) can also be proved without the help of the Baire Category Theorem as
follows. First one may assume that f 0, as the assumption on f is also
valid for the real and imaginary parts of f , and then for their positive and
negative parts. If (??) is Rnot valid, then for any k N, there exists gk with
||gk ||L2 () = 1 such that f gk d = ak k. One may even take gk 0.
Define
l
X
hl =
(kak )1 gk ,
k=1
Pl
P
then ||hl ||L2 () k=1 k , and f hl d = lk=1 k 1 . It now follows by
Monotone Convergence Theorem that
X
h=
(kak )1 gk L2 (),
2
k=1
f hd
11
f gd = , a contradiction.
6. Let x = (x1 , x2 ) and y = (y1 , y2 ) be vectors over the field F = Z/3. Show
that the bilinear forms B(x, y) = x1 y1 x2 y2 and D(x, y) = x1 y1 + x2 y2
are equivalent.
Solution. Interpreting x and y as column vectors, then D(x, y) = xt y and
B(x, y) = xt (I)y. Thus we need a matrix
a b
P =
c d
with coefficients in F such that P t (I)P = I, or P t P = I. Thus entries of
P must satisfy a2 + c2 = 1, b2 + d2 = 1, and ab + cd = 0. One solution is
a = b = d = 1 and c = 1, since 2 = 1 in F .
7. Consider the curve S = {(x, sin(1/x)) : x (0, 1]} R2 . Let T =
S ({0} [1, 1]). Show that T is a connected subset of R2 .
Solution. Suppose A, B is a pair of disjoint non-empty open sets of R2
whose union contains S. S is the image of the connected set (0, 1] under the
continuous map f : (0, 1] R2 given by f (x) = (x, sin(1/x)) and is hence
connected. Thus S is a subset of A or B; assume S A. Every point (0, y)
of {0} [1, 1] = T S is a limit point of S. Indeed, let b = arcsin(y) and
for k a positive integer let xk = 1/(b + 2k). Then f (xk ) = (xk , y), which
converges to (0, y) as k . So if (0, y) B then B contains points of S
since B is open. This contradicts the assumption A B = , so we conclude
that T A, and hence T is connected.
8. Let G be a finite group. Prove that G is cyclic if and only if G has exactly
one subgroup of order n for each positive integer n dividing |G|.
Solution. Let N = |G|. If G is a cyclic group then G
= Z/N Z. The
subgroups of G are in one-to-one correspondence with subgroups kZ of Z
containing N Z, i.e., with k | N , by the isomorphism theorems. Since
kZ/N Z
= Z/(N/k)Z there is exactly one subgroup of order n = N/k for
each n dividing N .
12
iz
.
i+z
Since fractional linear transformations carry circles (a line being a circle
through ) to circles, and g has the values g(i) = 0, g(0) = 1, g(i) = ,
and g(1) = i, it follows that g maps the imaginary axis to the real axis, and
maps the unit circle to the imaginary axis. Also
g(z) =
g(iy) =
1y
,
1+y
so g maps {iy : 1 < y < 1} to the positive real axis. Since 1/2 U
and g(1/2) = (3 + 4i)/5 is in the first quadrant, it follows that g maps U
conformally to the first quadrant
Q = {x + iy C : x > 0 and y > 0} .
Next, the map z 7 z 2 sends Q conformally to the upper half-plane
H = {x + iy C : y > 0} .
13
Finally, if we let
zi
,
z+i
then h maps H to D, because |h(z)| = 1 if z is real (since in that case
h(z) = w/w if w = z i), and h(i) = 0. So, the composite map f given by
i z 2
i z 2
2
f (z) = h(g(z) ) =
i
+i
i+z
i+z
h(z) =